LSAT and Law School Admissions Forum

Get expert LSAT preparation and law school admissions advice from PowerScore Test Preparation.

User avatar
 Dave Killoran
PowerScore Staff
  • PowerScore Staff
  • Posts: 5853
  • Joined: Mar 25, 2011
|
#63879
Complete Question Explanation
(The complete setup for this game can be found here: lsat/viewtopic.php?t=15078)

The correct answer choice is (A)

Because this List question includes only the parking lots and not the costs, at first you may not think you have enough information to eliminate each of the four incorrect answer choices. However, two of the answer choices can be eliminated by the numerical distribution, and the other two answer choices can be eliminated by combining the rules.

Answer choice (A): This is the correct answer choice.

Answer choice (B): This is probably the most difficult answer choice to eliminate, and it relies on an inference that is very difficult to make during the setup.

In this answer choice, lot Z is listed on Wednesday. According to our analysis of Wednesday costs, this lot must cost either $12 or $15. In this same answer, lot X is listed on Friday, and from our analysis of Friday costs, this lot must cost either $10 or $12. This combination presents a problem because the second rule indicates that lot X must cost more than lot Z. If the most that lot X can cost is $12, and the least that lot Z can cost is $12, then there is no way to conform to the second rule, and this answer choice must be incorrect.

This relationship reveals the inference that Z cannot be the Wednesday lot and X cannot be the Friday lot in the same solution (Z could be the Wednesday lot if X is not the Friday lot, and X could be the Friday lot if Z is not the Wednesday lot).

Answer choice (C): Because this answer choice has an equal number of X and Z lots, it violates the last rule and is therefore incorrect.

Answer choice (D): The game scenario stipulates that Anastasia parks in each of three lots at least once during the week. This answer choice does not include lot Y, and therefore it is incorrect. Note that, whenever you have a distribution in a game, there is often a List question answer choice that will attempt to drop one of the variables that must be present.

Answer choice (E): As discussed in the setup, the combination of the first two rules yields the inference that Anastasia must park in lot X or Y on Thursday. Because this answer specifies that she parks in lot Z on Thursday, this answer is incorrect.

Get the most out of your LSAT Prep Plus subscription.

Analyze and track your performance with our Testing and Analytics Package.